Practice Data sufficiency - quantitative aptitude Online Quiz (set-1) For All Competitive Exams

Q-1)  

Directions:

Each of the questions below consists of a question and two statements numbered I and II given below it. You have to decide whether the data provided in the statements are sufficient to answer the question. Read both the statements and:
  1. If the data in Statement I alone is sufficient to answer the question, while the data in Statement II alone is not sufficient to answer the question.
  2. If the data in Statement II alone is sufficient to answer the question, while the data in Statement I alone is not sufficient to answer the question.
  3. If the data in Statement I alone or in Statement II alone is sufficient to answer the question.
  4. If the data in both the Statements I and II are not sufficient to answer the question.
  5. If the data in both the Statements I and II together are necessary to answer the question.

What is the father's present age?
Statements :
I. Father's present age is five times the son's present age.
II. Five years ago the father's age was fifteen times the son's age at that time.

(a)

(b)

(c)

(d)

Explanation:

Let the present age of father and son be x and y years respectively.

From statement I,

x = 5 × y ...(i)

From statement II,

x – 5 = 15 × (y – 5) ...(ii)

Solving for x and y we get x = 35. So both statements together are necessary to give the answer.


Q-2)  

Directions:

Each of the questions below consists of a question and two statements numbered I and II given below it. You have to decide whether the data provided in the statements are sufficient to answer the question. Read both the statements and:
  1. If the data in Statement I alone is sufficient to answer the question, while the data in Statement II alone is not sufficient to answer the question.
  2. If the data in Statement II alone is sufficient to answer the question, while the data in Statement I alone is not sufficient to answer the question.
  3. If the data in Statement I alone or in Statement II alone is sufficient to answer the question.
  4. If the data in both the Statements I and II are not sufficient to answer the question.
  5. If the data in both the Statements I and II together are necessary to answer the question.

How is 'cricket' written in the code language?
Statements :
I. 'Dinesh, play cricket' is written as 'do, si ha'.
II. 'play cricket now' is written as 'ha si ma''.

(a)

(b)

(c)

(d)

Explanation:

From statement I + II

Dinesh play cricket→ dosi ha

play cricket now →ha si ma

Hence the code for 'cricket' is either 'ha' or 'si'.


Q-3)  

Directions:

Each of the questions below consists of a question and two statements numbered I and II given below it. You have to decide whether the data provided in the statements are sufficient to answer the question. Read both the statements and:
  1. If the data in Statement I alone is sufficient to answer the question, while the data in Statement II alone is not sufficient to answer the question.
  2. If the data in Statement II alone is sufficient to answer the question, while the data in Statement I alone is not sufficient to answer the question.
  3. If the data in Statement I alone or in Statement II alone is sufficient to answer the question.
  4. If the data in both the Statements I and II are not sufficient to answer the question.
  5. If the data in both the Statements I and II together are necessary to answer the question.

How is Gita related to Ganesh ?
Statements :
I. Gita's brother is Ganesh's father's eldest son.
II. Ganesh's wife's mother-in-law is Gita's mother.

(a)

(b)

(c)

(d)

Explanation:

There is no information about Geeta's gender so relationship between Geeta & Ganesh cannot be determined.


Q-4)  

Directions:

Each of the questions below consists of a question and two statements numbered I and II given below it. You have to decide whether the data provided in the statements are sufficient to answer the question. Read both the statements and:
  1. If the data in Statement I alone is sufficient to answer the question, while the data in Statement II alone is not sufficient to answer the question.
  2. If the data in Statement II alone is sufficient to answer the question, while the data in Statement I alone is not sufficient to answer the question.
  3. If the data in Statement I alone or in Statement II alone is sufficient to answer the question.
  4. If the data in both the Statements I and II are not sufficient to answer the question.
  5. If the data in both the Statements I and II together are necessary to answer the question.

Which direction is Shilpa facing ?
Statements :
I. If Meeta who is currently facing South, and turns 90° towards her left, she would face the same direction as Shilpa.
II. If Uday who is currently facing North, turns 90° towards his left, he would face a direction just opposite to the direction Shilpa is facing.

(a)

(b)

(c)

(d)

Explanation:

From statement I ⇒Shilpa is facing East.

From statement II ⇒ Shilpa is facing East.


Q-5)  

Directions:

Each of the questions below consists of a question and two statements numbered I and II given below it. You have to decide whether the data provided in the statements are sufficient to answer the question. Read both the statements and:
  1. If the data in Statement I alone is sufficient to answer the question, while the data in Statement II alone is not sufficient to answer the question.
  2. If the data in Statement II alone is sufficient to answer the question, while the data in Statement I alone is not sufficient to answer the question.
  3. If the data in Statement I alone or in Statement II alone is sufficient to answer the question.
  4. If the data in both the Statements I and II are not sufficient to answer the question.
  5. If the data in both the Statements I and II together are necessary to answer the question.

Lalita is in which direction with respect to Sangita ?
Statements :
I. Lalita is to the East, of Prabha who is to the South of Sangita.
II. Vinita is to the North of Lalita who is to the East of Sangita.

(a)

(b)

(c)

(d)

Explanation:

From statement I,

data-sufficiency-mcq-problems-competitive-exams-quantitative-aptitude

Lalita is in South East direction of Sangeeta

From statement II,

Lalita is in east direction of Sangeeta


Q-6)  

Directions:

Each of the questions below consists of a question and two statements numbered I and II given below it. You have to decide whether the data provided in the statements are sufficient to answer the question. Read both the statements and:
  1. If the data in Statement I alone is sufficient to answer the question, while the data in Statement II alone is not sufficient to answer the question.
  2. If the data in Statement II alone is sufficient to answer the question, while the data in Statement I alone is not sufficient to answer the question.
  3. If the data in Statement I alone or in Statement II alone is sufficient to answer the question.
  4. If the data in both the Statements I and II are not sufficient to answer the question.
  5. If the data in both the Statements I and II together are necessary to answer the question.

How is J related to N ?
Statements :
I. R is the brother of J and N is the son of R.
II. J is the sister of R and N is the son of R.

(a)

(b)

(c)

(d)

Explanation:

From statement I and II

⇒ J is paternal aunt of N.


Q-7)  

Directions:

Each of the questions below consists of a question and two statements numbered I and II given below it. You have to decide whether the data provided in the statements are sufficient to answer the question. Read both the statements and:
  1. If the data in Statement I alone is sufficient to answer the question, while the data in Statement II alone is not sufficient to answer the question.
  2. If the data in Statement II alone is sufficient to answer the question, while the data in Statement I alone is not sufficient to answer the question.
  3. If the data in Statement I alone or in Statement II alone is sufficient to answer the question.
  4. If the data in both the Statements I and II are not sufficient to answer the question.
  5. If the data in both the Statements I and II together are necessary to answer the question.

Among M, N, T, R and D each having different age, who is the youngest?
Statements :
I. N is younger than only D among them.
II. T is older than R and younger than M.

(a)

(b)

(c)

(d)

Explanation:

From I ⇒ N is the youngest among them.


Q-8)  

Directions:

Each of the questions below consists of a question and two statements numbered I and II given below it. You have to decide whether the data provided in the statements are sufficient to answer the question. Read both the statements and:
  1. If the data in Statement I alone is sufficient to answer the question, while the data in Statement II alone is not sufficient to answer the question.
  2. If the data in Statement II alone is sufficient to answer the question, while the data in Statement I alone is not sufficient to answer the question.
  3. If the data in Statement I alone or in Statement II alone is sufficient to answer the question.
  4. If the data in both the Statements I and II are not sufficient to answer the question.
  5. If the data in both the Statements I and II together are necessary to answer the question.

What is the code for 'going' in the code language ?
Statements :
I. In the code language 'where are you going' is written as 'ma ka ta re'
II. In the code language 'going to college' is written as 'lo pe ta'

(a)

(b)

(c)

(d)

Explanation:

From statement I + II

where are you going ma ka ta re

going to collage lo pe ta


Q-9)  

Directions:

Each of the questions below consists of a question and two statements numbered I and II given below it. You have to decide whether the data provided in the statements are sufficient to answer the question. Read both the statements and:
  1. If the data in Statement I alone is sufficient to answer the question, while the data in Statement II alone is not sufficient to answer the question.
  2. If the data in Statement II alone is sufficient to answer the question, while the data in Statement I alone is not sufficient to answer the question.
  3. If the data in Statement I alone or in Statement II alone is sufficient to answer the question.
  4. If the data in both the Statements I and II are not sufficient to answer the question.
  5. If the data in both the Statements I and II together are necessary to answer the question.

How many children are there in the class ?
Statements :
I. If arranged in ascending order of height, Suma is tenth from the top.
II. In order of height, Suma is five positions above Ranjit who is eighth from the`bottom.

(a)

(b)

(c)

(d)

Explanation:

From statement I + II

Suma's rank from the top = 10th

Suma's rank from the bottom = 8 + 5 = 13th

Number of children are there in the class = (10 + 13) - 1 = 22


Q-10)  

Directions:

Each of the questions below consists of a question and two statements numbered I and II given below it. You have to decide whether the data provided in the statements are sufficient to answer the question. Read both the statements and:
  1. If the data in Statement I alone is sufficient to answer the question, while the data in Statement II alone is not sufficient to answer the question.
  2. If the data in Statement II alone is sufficient to answer the question, while the data in Statement I alone is not sufficient to answer the question.
  3. If the data in Statement I alone or in Statement II alone is sufficient to answer the question.
  4. If the data in both the Statements I and II are not sufficient to answer the question.
  5. If the data in both the Statements I and II together are necessary to answer the question.

How far does Shruti live from the school?
Statements :
I. Shruti has to cycle 3 kms to her friend Mina's house which is 4 kms from the school.
II. Gitanjali lives exactly opposite to the school and walks 2 kms to reach Shruti's house.

(a)

(b)

(c)

(d)

Explanation:

Statement I does not inform whether Mina's house is towards the school or in the other direction to Shruti's house.

From statement II, we can infer that Shruti lives 2 km from the school.


Q-11)  

Directions:

Each of the questions below consists of a question and two statements numbered I and II given below it. You have to decide whether the data provided in the statements are sufficient to answer the question. Read both the statements and:
  1. If the data in Statement I alone is sufficient to answer the question, while the data in Statement II alone is not sufficient to answer the question.
  2. If the data in Statement II alone is sufficient to answer the question, while the data in Statement I alone is not sufficient to answer the question.
  3. If the data in Statement I alone or in Statement II alone is sufficient to answer the question.
  4. If the data in both the Statements I and II are not sufficient to answer the question.
  5. If the data in both the Statements I and II together are necessary to answer the question.

On which day of the week does Arti's birthday fall ?
Statements :
I. Sonu correctly remembers that Arti's birthday falls after Wednesday but before Sunday.
II. Raj correctly remembers that Arti's birthday falls before Friday but after Tuesday.

(a)

(b)

(c)

(d)

Explanation:

From I ⇒ Thursday or Friday or Saturday

From II ⇒ Wednesday or Thursday

I + II⇒ Arti's birthday falls on Thursday.


Q-12)  

Directions:

Each question below is followed by two statements I and II. You are to determine whether the data given in the statements is sufficient for answering the question. You should use the data and your knowledge of Mathematics to choose between the possible answers.
  1. (a) if the statement I alone is sufficient to answer the question, but the statement II alone is not sufficient.
  2. (b) if the statement II alone is sufficient to answer the question, but the statement I alone is not sufficient.
  3. (c) if both statements I and II together are needed to answer the question.
  4. (d) if either the statement I alone or statement II alone is sufficient to answer the question.
  5. (e) if you cannot get the answer from the statement I and II together, but need even more data

In how many years can a simple interest of Rs. 6,570 be obtained on an amount of Rs. 36,500 ?
Statements :
I. The rate of simple interest is 6 p.c.p.a.
II. The difference between the simple interest and compound interest is Rs. 402.084.

(a)

(b)

(c)

(d)

Explanation:

From statement I

⇒ years = ${6570×100}/{36500×6}$=3 years

Statement II does not give the rate of interest & so cannot be used.


Q-13)  

Directions:

Each question below is followed by two statements I and II. You are to determine whether the data given in the statements is sufficient for answering the question. You should use the data and your knowledge of Mathematics to choose between the possible answers.
  1. (a) if the statement I alone is sufficient to answer the question, but the statement II alone is not sufficient.
  2. (b) if the statement II alone is sufficient to answer the question, but the statement I alone is not sufficient.
  3. (c) if both statements I and II together are needed to answer the question.
  4. (d) if either the statement I alone or statement II alone is sufficient to answer the question.
  5. (e) if you cannot get the answer from the statement I and II together, but need even more data

What is the number of teachers in the school?
Statements :
I. Each teacher takes at least three lectures in a day.
II. There are 45 lectures in a week.

(a)

(b)

(c)

(d)

Explanation:

The question cannot be answered even after using information of both statements together.


Q-14)  

Directions:

Each question below is followed by two statements I and II. You are to determine whether the data given in the statements is sufficient for answering the question. You should use the data and your knowledge of Mathematics to choose between the possible answers.
  1. (a) if the statement I alone is sufficient to answer the question, but the statement II alone is not sufficient.
  2. (b) if the statement II alone is sufficient to answer the question, but the statement I alone is not sufficient.
  3. (c) if both statements I and II together are needed to answer the question.
  4. (d) if either the statement I alone or statement II alone is sufficient to answer the question.
  5. (e) if you cannot get the answer from the statement I and II together, but need even more data

What is the area of the rectangular plot?
Statements :
I. The length of the plot is 375 metres.
II. The length of the plot is thrice it's breadth.

(a)

(b)

(c)

(d)

Explanation:

From statement I

⇒ Length = 375 metres

From II Breadth = $375/3$ = 125metres


Q-15)  

Directions:

Each question below is followed by two statements I and II. You are to determine whether the data given in the statements is sufficient for answering the question. You should use the data and your knowledge of Mathematics to choose between the possible answers.
  1. (a) if the statement I alone is sufficient to answer the question, but the statement II alone is not sufficient.
  2. (b) if the statement II alone is sufficient to answer the question, but the statement I alone is not sufficient.
  3. (c) if both statements I and II together are needed to answer the question.
  4. (d) if either the statement I alone or statement II alone is sufficient to answer the question.
  5. (e) if you cannot get the answer from the statement I and II together, but need even more data

What is the three digit number?
Statements :
I. The three digit number is an exact multiple of 13.
II. The first and the third digit of the number are 7.

(a)

(b)

(c)

(d)

Explanation:

From statement II⇒the numbers could be

707, 717, 727, 737, 747, 757, 767, 777, 787, 797.

From statement I, out of the 10 numbers only 767 is divisible by 13.


Q-16)  

Directions:

Each question below is followed by two statements I and II. You are to determine whether the data given in the statements is sufficient for answering the question. You should use the data and your knowledge of Mathematics to choose between the possible answers.
  1. (a) if the statement I alone is sufficient to answer the question, but the statement II alone is not sufficient.
  2. (b) if the statement II alone is sufficient to answer the question, but the statement I alone is not sufficient.
  3. (c) if both statements I and II together are needed to answer the question.
  4. (d) if either the statement I alone or statement II alone is sufficient to answer the question.
  5. (e) if you cannot get the answer from the statement I and II together, but need even more data

What is the salary of A, in a group of A, B, C, D and E, whose average salary is Rs. 65,780?
Statements :
I. Total of the salary of B and C is Rs. 88,545.
II. Total of the salary of D and E is Rs. 59,020.

(a)

(b)

(c)

(d)

Explanation:

From statement I and II,

⇒ Salary of A

= 65780 × 5 – (88545 + 59020)

= 328900 – 147565 = Rs.181335


Q-17)  

Directions:

Each question below is followed by two statements I and II. You are to determine whether the data given in the statements is sufficient for answering the question. You should use the data and your knowledge of Mathematics to choose between the possible answers.
  1. (a) if the statement I alone is sufficient to answer the question, but the statement II alone is not sufficient.
  2. (b) if the statement II alone is sufficient to answer the question, but the statement I alone is not sufficient.
  3. (c) if both statements I and II together are needed to answer the question.
  4. (d) if either the statement I alone or statement II alone is sufficient to answer the question.
  5. (e) if you cannot get the answer from the statement I and II together, but need even more data

What is the age of C, in a group of A, B, C, D and E, whose average age is 45 years?
I. Average of the ages of A and B is 53 years.
II.Average of the ages of D and E is 47 years

(a)

(b)

(c)

(d)

(e)

Explanation:

Age of C = Total age – age of (A + B + D + E)

= Total age – 2 × Average ages of (A + B) – 2 × Average ages of (D + E)


Q-18)  

Directions:

Each question below is followed by two statements I and II. You are to determine whether the data given in the statements is sufficient for answering the question. You should use the data and your knowledge of Mathematics to choose between the possible answers.
  1. (a) if the statement I alone is sufficient to answer the question, but the statement II alone is not sufficient.
  2. (b) if the statement II alone is sufficient to answer the question, but the statement I alone is not sufficient.
  3. (c) if both statements I and II together are needed to answer the question.
  4. (d) if either the statement I alone or statement II alone is sufficient to answer the question.
  5. (e) if you cannot get the answer from the statement I and II together, but need even more data

Towards which direction was D facing when he started his journey?
I. D walked 20 metres after he started, took a right turn and walked 30 metres and again took a right turn and faced West.
II.D walked 20 metres after he started, took a left turn and walked 30 metres and again took a left turn and faced West.

(a)

(b)

(c)

(d)

(e)

Explanation:

data-sufficiency-mcq-problems-competitive-exams-quantitative-aptitude

D was facing East when he started his journey, from statement II.


Q-19)  

Directions:

Each question below is followed by two statements I and II. You are to determine whether the data given in the statements is sufficient for answering the question. You should use the data and your knowledge of Mathematics to choose between the possible answers.
  1. (a) if the statement I alone is sufficient to answer the question, but the statement II alone is not sufficient.
  2. (b) if the statement II alone is sufficient to answer the question, but the statement I alone is not sufficient.
  3. (c) if both statements I and II together are needed to answer the question.
  4. (d) if either the statement I alone or statement II alone is sufficient to answer the question.
  5. (e) if you cannot get the answer from the statement I and II together, but need even more data

How is Anil related to Sanjay?
I. Sanjay's son is the brother of only sister of Anil.
II.Radhika, the only daughter of Sanjay has only two brothers.

(a)

(b)

(c)

(d)

(e)

Explanation:


Q-20)  

Directions:

Each question below is followed by two statements I and II. You are to determine whether the data given in the statements is sufficient for answering the question. You should use the data and your knowledge of Mathematics to choose between the possible answers.
  1. (a) if the statement I alone is sufficient to answer the question, but the statement II alone is not sufficient.
  2. (b) if the statement II alone is sufficient to answer the question, but the statement I alone is not sufficient.
  3. (c) if both statements I and II together are needed to answer the question.
  4. (d) if either the statement I alone or statement II alone is sufficient to answer the question.
  5. (e) if you cannot get the answer from the statement I and II together, but need even more data

How many daughters does P have?
I. K and M are sisters of T.
II.T's father is husband of P's mother.

(a)

(b)

(c)

(d)

(e)

Explanation:

From both the statements K, M and T are siblings of P